Last visit was: 26 Apr 2024, 19:20 It is currently 26 Apr 2024, 19:20

Close
GMAT Club Daily Prep
Thank you for using the timer - this advanced tool can estimate your performance and suggest more practice questions. We have subscribed you to Daily Prep Questions via email.

Customized
for You

we will pick new questions that match your level based on your Timer History

Track
Your Progress

every week, we’ll send you an estimated GMAT score based on your performance

Practice
Pays

we will pick new questions that match your level based on your Timer History
Not interested in getting valuable practice questions and articles delivered to your email? No problem, unsubscribe here.
Close
Request Expert Reply
Confirm Cancel
SORT BY:
Date
Tags:
Show Tags
Hide Tags
User avatar
Manager
Manager
Joined: 05 Jan 2011
Posts: 73
Own Kudos [?]: 956 [57]
Given Kudos: 8
Send PM
Most Helpful Reply
User avatar
Manager
Manager
Joined: 03 Mar 2011
Posts: 58
Own Kudos [?]: 252 [16]
Given Kudos: 12
Location: United States
Schools: Erasmus (S)
GMAT 1: 730 Q51 V37
GPA: 3.9
Send PM
Retired Moderator
Joined: 16 Nov 2010
Posts: 909
Own Kudos [?]: 1173 [12]
Given Kudos: 43
Location: United States (IN)
Concentration: Strategy, Technology
Send PM
General Discussion
User avatar
Manager
Manager
Joined: 26 Sep 2013
Posts: 151
Own Kudos [?]: 598 [0]
Given Kudos: 40
Concentration: Finance, Economics
GMAT 1: 670 Q39 V41
GMAT 2: 730 Q49 V41
Send PM
Re: n is an integer greater than or equal to 0. The sequence tn [#permalink]
It should be E...n-1 being divisible by 4, based on the chart, that means it's either 4 (as you can see if it's 4, then the next term in the sequence is 6), or 24 for (in which case the next term in the series is 31). OA is incorrect.
Math Expert
Joined: 02 Sep 2009
Posts: 92948
Own Kudos [?]: 619249 [12]
Given Kudos: 81609
Send PM
Re: n is an integer greater than or equal to 0. The sequence tn [#permalink]
6
Kudos
6
Bookmarks
Expert Reply
AccipiterQ wrote:
n is an integer greater than or equal to 0. The sequence \(t_n\) for n > 0 is defined as \(t_n = t_{n-1} + n\). Given that \(t_0 = 3\), is tn even?

(1) n + 1 is divisible by 3
(2) n - 1 is divisible by 4

It should be E...n-1 being divisible by 4, based on the chart, that means it's either 4 (as you can see if it's 4, then the next term in the sequence is 6), or 24 for (in which case the next term in the series is 31). OA is incorrect.


The OA is correct.

(2) n - 1 is divisible by 4 means that \(n=4k+1\), thus n is 1, 5, 9, ...

\(t_1=4=even\).
\(t_5=18=even\).
\(t_9=48=even\).
...

All are even.
User avatar
Manager
Manager
Joined: 04 Oct 2013
Posts: 141
Own Kudos [?]: 584 [5]
Given Kudos: 29
Concentration: Finance, Leadership
GMAT 1: 590 Q40 V30
GMAT 2: 730 Q49 V40
WE:Project Management (Entertainment and Sports)
Send PM
Re: n is an integer greater than or equal to 0. The sequence tn [#permalink]
5
Kudos
First off let's see the sequence behavior charting some numbers.

t(0)=3+0 = O
t(1)=3+1 = E
t(2)=4+2 = E
t(3)=6+3 = O
t(4)=9+4 = O
t(5)=13+5= E
t(8) = O

we can notice a repeating pattern (E, E, O, O) we need to figure out how n relates to a multiple of 4.

st1 n could be 2, 5, 8, 11, 14 etc.. checking the chart we can tell that this statement is not sufficient
st2 tells us how n relates to a multiple of 4 and indeed if we plug some values in we can safely claim that t(n) is even.
User avatar
Intern
Intern
Joined: 30 Sep 2013
Status:Yes, I can and I will. 700+FTW
Posts: 20
Own Kudos [?]: 131 [0]
Given Kudos: 3
Location: India
Concentration: Finance, Strategy
GMAT Date: 02-05-2014
GPA: 3.7
WE:Other (Retail)
Send PM
Re: n is an integer greater than or equal to 0. The sequence tn [#permalink]
I have a doubt here, 2nd part says that it is divisible by 4 then why have you wrote the expression as n= 4k+1. it doesn't talks about remainder here right? isn't n=4k enough?

Bunuel wrote:
AccipiterQ wrote:
n is an integer greater than or equal to 0. The sequence \(t_n\) for n > 0 is defined as \(t_n = t_{n-1} + n\). Given that \(t_0 = 3\), is tn even?

(1) n + 1 is divisible by 3
(2) n - 1 is divisible by 4

It should be E...n-1 being divisible by 4, based on the chart, that means it's either 4 (as you can see if it's 4, then the next term in the sequence is 6), or 24 for (in which case the next term in the series is 31). OA is incorrect.


The OA is correct.

(2) n - 1 is divisible by 4 means that \(n=4k+1\), thus n is 1, 5, 9, ...

\(t_1=4=even\).
\(t_5=18=even\).
\(t_9=48=even\).
...

All are even.
Math Expert
Joined: 02 Sep 2009
Posts: 92948
Own Kudos [?]: 619249 [0]
Given Kudos: 81609
Send PM
Re: n is an integer greater than or equal to 0. The sequence tn [#permalink]
Expert Reply
rgyanani wrote:
I have a doubt here, 2nd part says that it is divisible by 4 then why have you wrote the expression as n= 4k+1. it doesn't talks about remainder here right? isn't n=4k enough?

Bunuel wrote:
AccipiterQ wrote:
n is an integer greater than or equal to 0. The sequence \(t_n\) for n > 0 is defined as \(t_n = t_{n-1} + n\). Given that \(t_0 = 3\), is tn even?

(1) n + 1 is divisible by 3
(2) n - 1 is divisible by 4

It should be E...n-1 being divisible by 4, based on the chart, that means it's either 4 (as you can see if it's 4, then the next term in the sequence is 6), or 24 for (in which case the next term in the series is 31). OA is incorrect.


The OA is correct.

(2) n - 1 is divisible by 4 means that \(n=4k+1\), thus n is 1, 5, 9, ...

\(t_1=4=even\).
\(t_5=18=even\).
\(t_9=48=even\).
...

All are even.


n - 1 is divisible by 4 --> \(n-1=4k\) --> \(n=4k+1\) --> n is 1 more than a multiple of 4.
Intern
Intern
Joined: 09 Nov 2013
Posts: 48
Own Kudos [?]: 33 [1]
Given Kudos: 32
Send PM
Re: n is an integer greater than or equal to 0. The sequence tn [#permalink]
1
Kudos
Hi
Bunuel pl review the logic below

its a AP where d = tn-tn-1= n where 3 is the first term

Now the tn = 3+ (n-1)n {by formula tn= a+(n-1)d}

in this equation n and n-1 are consecutive numbers

for statement #1: n+1, which is next consecutive number in the sequence, is divisible by 3, but we don't know whether its even or odd ( including 3,6,9,..) so insuff

for statement #2: n-1 is divisible by 4 so n-1 is even hence n is odd and n(n-1) is even. And 3 + even = odd suff

thanks
sid
Intern
Intern
Joined: 23 Mar 2015
Posts: 5
Own Kudos [?]: [0]
Given Kudos: 5
Send PM
Re: n is an integer greater than or equal to 0. The sequence tn [#permalink]
Hi Bunnel,

If we apply the nth term of an AP formula,
we have
tn= 3 + (n-1)* d
In this AP, (this is an AP because tn -tn-1=n)
we get, tn=3 + (n-1)*n
now because (n-1)*n is always even (product of two consecutive numbers is always even), we get
tn= 3 + even = odd.
So clearly, tn is odd so (A) should be sufficient.

Moving to B, (n-1) is divisible by 4, that means n-1 is even so n is odd,
again tn = 3+n-1*n = 3+ even = odd

Why is th e answer not D?
Math Expert
Joined: 02 Sep 2009
Posts: 92948
Own Kudos [?]: 619249 [1]
Given Kudos: 81609
Send PM
Re: n is an integer greater than or equal to 0. The sequence tn [#permalink]
1
Kudos
Expert Reply
gauravprashar17 wrote:
Hi Bunnel,

If we apply the nth term of an AP formula,
we have
tn= 3 + (n-1)* d
In this AP, (this is an AP because tn -tn-1=n)
we get, tn=3 + (n-1)*n
now because (n-1)*n is always even (product of two consecutive numbers is always even), we get
tn= 3 + even = odd.
So clearly, tn is odd so (A) should be sufficient.

Moving to B, (n-1) is divisible by 4, that means n-1 is even so n is odd,
again tn = 3+n-1*n = 3+ even = odd

Why is th e answer not D?


The given sequence is NOT an arithmetic progression. You'd notice it if you'd try to write down several terms:
\(t_0=3\)
\(t_1=t_0+1=4\)
\(t_2=t_1+2=6\)
\(t_3=t_2+3=9\)
\(t_4=t_3+4=13\)
\(t_5=t_4+5=18\)
\(t_6=t_5+6=24\)
\(t_7=t_6+7=31\)
\(t_8=t_7+8=39\)

...

(1) n + 1 is divisible by 3 --> n + 1 = 3x --> n = 3x - 1, thus n can be 2, 5, 8, ... tn ca be even (for example, t2) and odd (for example, t8).
Manager
Manager
Joined: 05 Dec 2015
Posts: 82
Own Kudos [?]: 16 [0]
Given Kudos: 982
Send PM
Re: n is an integer greater than or equal to 0. The sequence tn [#permalink]
Bunuel please help:

I get why (1) is NS:
- T_0 => yes N even
- T_1 => No N not even

I'm confused on (2):

- if T_7 then no N = odd
- if T_2 then yes N = odd

where is my logic off? Thanks!!

Bunuel wrote:
gauravprashar17 wrote:
Hi Bunnel,

If we apply the nth term of an AP formula,
we have
tn= 3 + (n-1)* d
In this AP, (this is an AP because tn -tn-1=n)
we get, tn=3 + (n-1)*n
now because (n-1)*n is always even (product of two consecutive numbers is always even), we get
tn= 3 + even = odd.
So clearly, tn is odd so (A) should be sufficient.

Moving to B, (n-1) is divisible by 4, that means n-1 is even so n is odd,
again tn = 3+n-1*n = 3+ even = odd

Why is th e answer not D?


The given sequence is NOT an arithmetic progression. You'd notice it if you'd try to write down several terms:
\(t_0=3\)
\(t_1=t_0+1=4\)
\(t_2=t_1+2=6\)
\(t_3=t_2+3=9\)
\(t_4=t_3+4=13\)
\(t_5=t_4+5=18\)
\(t_6=t_5+6=24\)
\(t_7=t_6+7=31\)
\(t_8=t_7+8=39\)

...

(1) n + 1 is divisible by 3 --> n + 1 = 3x --> n = 3x - 1, thus n can be 2, 5, 8, ... tn ca be even (for example, t2) and odd (for example, t8).
Math Expert
Joined: 02 Sep 2009
Posts: 92948
Own Kudos [?]: 619249 [1]
Given Kudos: 81609
Send PM
Re: n is an integer greater than or equal to 0. The sequence tn [#permalink]
1
Bookmarks
Expert Reply
mdacosta wrote:
Bunuel please help:

I get why (1) is NS:
- T_0 => yes N even
- T_1 => No N not even

I'm confused on (2):

- if T_7 then no N = odd
- if T_2 then yes N = odd

where is my logic off? Thanks!!

Bunuel wrote:
gauravprashar17 wrote:
Hi Bunnel,

If we apply the nth term of an AP formula,
we have
tn= 3 + (n-1)* d
In this AP, (this is an AP because tn -tn-1=n)
we get, tn=3 + (n-1)*n
now because (n-1)*n is always even (product of two consecutive numbers is always even), we get
tn= 3 + even = odd.
So clearly, tn is odd so (A) should be sufficient.

Moving to B, (n-1) is divisible by 4, that means n-1 is even so n is odd,
again tn = 3+n-1*n = 3+ even = odd

Why is th e answer not D?


The given sequence is NOT an arithmetic progression. You'd notice it if you'd try to write down several terms:
\(t_0=3\)
\(t_1=t_0+1=4\)
\(t_2=t_1+2=6\)
\(t_3=t_2+3=9\)
\(t_4=t_3+4=13\)
\(t_5=t_4+5=18\)
\(t_6=t_5+6=24\)
\(t_7=t_6+7=31\)
\(t_8=t_7+8=39\)

...

(1) n + 1 is divisible by 3 --> n + 1 = 3x --> n = 3x - 1, thus n can be 2, 5, 8, ... tn ca be even (for example, t2) and odd (for example, t8).


(2) n - 1 is divisible by 4 means that \(n=4k+1\), thus n is 1, 5, 9, ... So, n cannot be 2 or 7.

\(t_1=4=even\).
\(t_5=18=even\).
\(t_9=48=even\).
...

All are even.
Senior Manager
Senior Manager
Joined: 05 Jan 2017
Posts: 416
Own Kudos [?]: 284 [1]
Given Kudos: 15
Location: India
Send PM
Re: n is an integer greater than or equal to 0. The sequence tn [#permalink]
1
Kudos
PROMPT ANALYSIS
n is a natural number and tn follows the sequence as tn = tn-1 + n with t0 = 3.
Therefore, t1 =4,t2 =6, t3=9,t4 =13 t5 = 18 t6 = 24 t7 = 31 t8 = 39 t9= 48 t10 =58 t11 = 69…..

Superset
The answer will be either YES or NO.

Translation
In order to know if tn is even we need:
1# exact value of n
2# any equation that will conclude the question

Statement analysis

St 1: n +1 = 3k; n = 3k-1. So we are talking about t2(even), t5(even), t8(odd), t11(odd). NO CONCLUSION. Hence option a, d eliminated.

St 2: n - 1 = 4j; n = 4j +1. So we are talking about t1(even), t5(even), t9(even) t13(even). Hence it is even. Answr.

Option B
User avatar
Non-Human User
Joined: 09 Sep 2013
Posts: 32690
Own Kudos [?]: 822 [0]
Given Kudos: 0
Send PM
Re: n is an integer greater than or equal to 0. The sequence tn [#permalink]
Hello from the GMAT Club BumpBot!

Thanks to another GMAT Club member, I have just discovered this valuable topic, yet it had no discussion for over a year. I am now bumping it up - doing my job. I think you may find it valuable (esp those replies with Kudos).

Want to see all other topics I dig out? Follow me (click follow button on profile). You will receive a summary of all topics I bump in your profile area as well as via email.
GMAT Club Bot
Re: n is an integer greater than or equal to 0. The sequence tn [#permalink]
Moderator:
Math Expert
92948 posts

Powered by phpBB © phpBB Group | Emoji artwork provided by EmojiOne